0
$\begingroup$

I have the following nonlinear optimisation problem arising in my model.

$$\min \sum_{k=0}^{N-1} (\tau-t_k)^+\quad \text{ s.t. } {\mathbf{x}^\top\mathbf{w}\le W,\ \mathbf{x}\ge0}, t_k=t_{k-1}+x_k \text{ and } t_0=0.$$

I tried to simplify this problem by setting $\lambda_k=(\tau-t_k)^+$. I get the condition that $\rho w_i=\frac{N-i}{K_i}+\mu_i $, for all $i$, where $\rho$ and $\mu_i$ are appropriate Lagrange multipliers. How do I use this to find the optimal solution?

$\endgroup$
1
  • $\begingroup$ Are $t,x$ the problem variables? And I guess $(x)^+= \max(x,0)$, right? $\endgroup$ Jan 1, 2016 at 14:35

1 Answer 1

2
$\begingroup$

Your problem is a linear problem: introducing auxiliary variables $z_k$ for $k=0,\ldots,,N-1$, you obtain

$$ \begin{align} &\min \sum_{k=0}^{N-1}z_k \\ &t_0 = 0\\ &t_{k} = t_{k-1} + x_k, \quad k=0,\ldots,N-1\\ &w^T x \leq W \\ &z_k\geq 0, \quad k=0,\ldots,N-1\\ &z_k\geq \tau - t_k, \quad k=0,\ldots,N-1\\ &x\geq 0 \end{align} $$

Any LP solver will do.

$\endgroup$

Your Answer

By clicking “Post Your Answer”, you agree to our terms of service and acknowledge you have read our privacy policy.

Not the answer you're looking for? Browse other questions tagged or ask your own question.